K
Khách

Hãy nhập câu hỏi của bạn vào đây, nếu là tài khoản VIP, bạn sẽ được ưu tiên trả lời.

22 tháng 1 2017

Ta có: \(\frac{x}{2x+y}+\frac{y}{2y+z}+\frac{z}{2z+x}\)

\(=\frac{1}{2}-\frac{y}{4x+2y}+\frac{1}{2}-\frac{z}{4y+2z}+\frac{1}{2}-\frac{x}{4z+2x}\)

\(=\frac{3}{2}-\left(\frac{y^2}{4xy+2y^2}+\frac{z^2}{4yz+2z^2}+\frac{x^2}{4zx+2x^2}\right)\)

\(\le\frac{3}{2}-\frac{\left(x+y+z\right)^2}{2x^2+2y^2+2z^2+4xy+4yz+4zx}\)

\(=\frac{3}{2}-\frac{\left(x+y+z\right)^2}{2\left(x+y+z\right)^2}=\frac{3}{2}-\frac{1}{2}=1\)

áp dụng bđt schwarts ta có:

\(\frac{1}{2x+1}+\frac{1}{2y+1}+\frac{1}{2z+1}\ge\frac{\left(1+1+1\right)^2}{2x+2y+2z+3}\ge\frac{9}{7}\)

\(\Rightarrow1-\frac{1}{2x+1}+1-\frac{1}{2y+1}+1-\frac{1}{2z+1}\le3-\frac{9}{7}\)

\(\Rightarrow\frac{2x}{2x+1}+\frac{2y}{2y+1}+\frac{2z}{2z+1}\le\frac{12}{7}\)

\(\Rightarrow\frac{x}{2x+1}+\frac{y}{2y+1}+\frac{z}{2z+1}\le\frac{6}{7}\left(Q.E.D\right)\)

dấu = xảy ra khi x=y=z=2/3

27 tháng 5 2020

Đặt \(\hept{\begin{cases}\sqrt{x}=p\\\sqrt{y}=q\\\sqrt{z}=r\end{cases}}\). Khi đó \(\hept{\begin{cases}p+q+r=1\\p,q,r>0\end{cases}}\)

và ta cần chứng minh \(\frac{pq}{\sqrt{p^2+q^2+2r^2}}+\frac{qr}{\sqrt{q^2+r^2+2p^2}}+\frac{rp}{\sqrt{r^2+p^2+2q^2}}\le\frac{1}{2}\)

Ta có: \(\frac{pq}{\sqrt{p^2+q^2+2r^2}}=\frac{2pq}{\sqrt{\left(1+1+2\right)\left(p^2+q^2+2r^2\right)}}\)

\(\le\frac{2pq}{p+q+2r}\le\frac{1}{2}\left(\frac{pq}{p+r}+\frac{pq}{q+r}\right)\)(Theo BĐT Cauchy-Schwarz và BĐT \(\frac{1}{u}+\frac{1}{v}\ge\frac{4}{u+v}\)) (1)

Hoàn toàn tương tự: \(\frac{qr}{\sqrt{q^2+r^2+2p^2}}\le\frac{1}{2}\left(\frac{qr}{q+p}+\frac{qr}{r+p}\right)\)(2); \(\frac{rp}{\sqrt{r^2+p^2+2q^2}}\le\frac{1}{2}\left(\frac{rp}{r+q}+\frac{rp}{p+q}\right)\)(3)

Cộng theo vế của 3 BĐT (1), (2), (3), ta được: \(\frac{pq}{\sqrt{p^2+q^2+2r^2}}+\frac{qr}{\sqrt{q^2+r^2+2p^2}}+\frac{rp}{\sqrt{r^2+p^2+2q^2}}\)\(\le\frac{1}{2}\left(\frac{r\left(p+q\right)}{p+q}+\frac{p\left(q+r\right)}{q+r}+\frac{q\left(r+p\right)}{r+p}\right)=\frac{1}{2}\left(p+q+r\right)=\frac{1}{2}\)(Do p + q + r = 1)

Đẳng thức xảy ra khi \(p=q=r=\frac{1}{3}\)hay \(x=y=z=\frac{1}{9}\)

26 tháng 10 2016

Ta có:

\(\frac{1}{x+y}+\frac{1}{y+z}+\frac{1}{z+x}=6\ge\frac{9}{2\left(x+y+z\right)}\)\(\Rightarrow x+y+z\ge\frac{3}{4}\)

Lại có: \(\frac{1}{2x+3y+3z}=\frac{\left(\frac{3}{4}+\frac{1}{4}\right)^2}{2\left(x+y+z\right)+y+z}\le\frac{9}{32\left(x+y+z\right)}+\frac{1}{16\left(y+z\right)}\)

Do đó:

\(\frac{1}{2x+3y+3z}+\frac{1}{2y+3x+3z}+\frac{1}{2z+3x+3y}\)

\(\le\frac{9}{32\left(x+y+z\right)}\cdot3+\frac{1}{16}\left(\frac{1}{x+y}+\frac{1}{y+z}+\frac{1}{z+x}\right)\)

\(\le\frac{9}{32\cdot\frac{3}{4}}+\frac{1}{16}\cdot6=\frac{3}{2}\)(Đpcm)

1 tháng 2 2018

Tại sao \(\frac{1}{x+y}+\frac{1}{y+z}+\frac{1}{x+z}=6\ge\frac{9}{2\left(x+y+z\right)}\)

NV
23 tháng 11 2019

a/ \(\frac{2x+1}{\sqrt{x^2+2}}+\left(x+1\right)\left(\sqrt{1+\frac{2x+1}{x^2+2}}-1\right)+2x+1=0\)

\(\Leftrightarrow\frac{2x+1}{\sqrt{x^2+2}}+\frac{\left(x+1\right)\left(2x+1\right)}{\sqrt{1+\frac{2x+1}{x^2+2}}+1}+2x+1=0\)

\(\Leftrightarrow\left(2x+1\right)\left(\frac{1}{\sqrt{x^2+2}}+\frac{x+1}{\sqrt{1+\frac{2x+1}{x^2+2}}+1}+1\right)=0\)

\(\Rightarrow x=-\frac{1}{2}\)

b/ \(Q\ge\frac{\left(x+y+z\right)^2}{xyz\left(x+y+z\right)}+\frac{\left(x^3+y^3+z^3\right)^2}{xy+yz+zx}\ge\frac{x+y+z}{xyz}+\frac{\left(x^2+y^2+z^2\right)^3}{\left(x+y+z\right)^2}\)

\(Q\ge\frac{27\left(x+y+z\right)}{\left(x+y+z\right)^3}+\frac{\left(x+y+z\right)^6}{27\left(x+y+z\right)^2}=\frac{27}{\left(x+y+z\right)^2}+\frac{\left(x+y+z\right)^4}{27}\)

\(Q\ge\frac{27}{64\left(x+y+z\right)^2}+\frac{27}{64\left(x+y+z\right)^2}+\frac{\left(x+y+z\right)^4}{27}+\frac{837}{32\left(x+y+z\right)^2}\)

\(Q\ge3\sqrt[3]{\frac{27^2\left(x+y+z\right)^4}{64^2.27\left(x+y+z\right)^4}}+\frac{837}{32.\left(\frac{3}{2}\right)^2}=\frac{195}{16}\)

"=" \(\Leftrightarrow x=y=z=\frac{1}{2}\)

23 tháng 11 2019

Nguyễn Trúc Giang, Duy Khang, Vũ Minh Tuấn, Võ Hồng Phúc, tth, No choice teen, Phạm Lan Hương,

Nguyễn Lê Phước Thịnh, @Nguyễn Việt Lâm, @Akai Haruma

giúp em vs ạ! Cần trước 5h chiều nay ạ

Thanks nhiều

24 tháng 4 2020

Bài 1 : 

Bât đẳng thức cần chứng minh tương đương với :

( xy+yz + zx )(9 + x2y2 +z2y2 + x2z2 ) \(\ge\)36xyz 

Áp dụng bất đẳng thức Côsi ta có : 

xy+ yz + zx \(\ge3\sqrt[3]{x^2y^2z^2}\)           ( 1) 

Và 9 + x2y2 + z2y2 + x2z2 \(\ge12\sqrt[12]{x^4y^4z^4}\)

hay 9+ x2y2 + z2y2+ x2z2 \(\ge12\sqrt[3]{xyz}\)                (2) 

Do các vế đều dương ,từ (1) và (2) suy ra :

( xy + yz +zx )( 9+ x2y2 + z2y2 + x2z2 ) \(\ge36xyz\left(đpcm\right)\)

Dấu đẳng thức xảy ra khi và chỉ khi x = y  =z = 1 

Bài 2: 

\(\hept{\begin{cases}a;b;c>0\\ab+bc+ca=1\end{cases}}\)

Có : \(\hept{\begin{cases}\sqrt{1+a^2}\ge\sqrt{2a}\Rightarrow\frac{a}{\sqrt{1+a^2}}\le\frac{\sqrt{3}}{2}a\\\sqrt{1+b^2}\ge\sqrt{2b}\Rightarrow\frac{b}{\sqrt{1+b^2}}\le\frac{\sqrt{3}}{2}b\\\sqrt{1+c^2}\ge\sqrt{2c}\Rightarrow\frac{c}{\sqrt{1+c^2}}\le\frac{\sqrt{3}}{2}c\end{cases}}\)

=> \(\sqrt{1+a^2}+\sqrt{1+b^2}+\sqrt{1+c^2}\le\frac{\sqrt{3}}{2}\left(a+b+c\right)\le\frac{\sqrt{3}}{2}.\frac{\sqrt{3}}{2}\left(ab+bc+ca\right)\)

=> \(\sqrt{1+a^2}+\sqrt{1+b^2}+\sqrt{1+c^2}\le\frac{3}{2}\left(đpcm\right)\)

Dấu "=" xảy ra khi và chỉ khi a =b =c = \(\frac{1}{\sqrt{3}}\)

16 tháng 4 2016

áp dụng bđt 1/a +1/b >= 4/(a+b) ta đc :....1/ (2x+y+z) <= 1/4(x+y)+1/4(x+z) ; 1/(2y+z+x)<=1/4(y+z)+1/4(x+y) ; 1/(2z+x+y)<=1/4(z+x)+1/4(z+y)

suy ra A (biểu thức đã cho ) <= 1/2(x+y) +1/2(y+z) +1/2(z+x)<= 1/8 . (1/x+1/y) +1/8. (1/y+1/z)+1/8(1/z+1/x) =1/8 . 2. (1/x+1/y+1/z)=1    (áp dụng lại bđt trên)...,suyra đpcm.

dấu ''='' xảy ra <=> x=y=z

25 tháng 10 2015

câu hỏi tương tự nhé bạn